LSAT and Law School Admissions Forum

Get expert LSAT preparation and law school admissions advice from PowerScore Test Preparation.

 Administrator
PowerScore Staff
  • PowerScore Staff
  • Posts: 8917
  • Joined: Feb 02, 2011
|
#25017
Complete Question Explanation

Parallel Reasoning—SN. The correct answer choice is (D)

The journalist concludes that the union members will go on strike. The author supports the conclusion by stating that independent arbitration would avert a strike, but only if both sides agreed to abide by the recommendation. We can diagram this relationship as follows:
  • Sufficient ..... ..... ..... ..... ..... ..... ..... Necessary

    Indep. arbitration can stop strike ..... :arrow: ..... Both sides agree to abide by ruling

The journalist states that the union is unlikely to agree to binding arbitration, so there is likely to be a strike. This is conditional reasoning, but contains two additional important features: 1) It predicts future actions of the union based on past actions; and 2) It is phrased in terms of likelihood, not certainty. We can diagram this relationship as follows:
  • Sufficient ..... ..... ..... ..... ..... ..... ..... Necessary

    Both sides agree to abide by ruling ..... :arrow: ..... Indep. arbitration can stop strike


This is the contrapositive of the conditional reasoning in the premise, and therefore it is valid reasoning. The correct answer choice also needs to include valid conditional reasoning using a statement and its contrapositive.

Answer choice (A): It would be helpful to diagram this answer choice in order to see if it matches the stimulus. The first sentence contains an “unless” condition, and can be diagramed as follows:
  • Sufficient ..... ..... ..... ..... Necessary

    Company downsize ..... :arrow: ..... More stock issued
The second sentence is also conditional, and begins with “if the company downsizes,” and contains a third conditional term—shareholders demanding change. This does not match the reasoning in the stimulus, so we can eliminate the answer choice and move on.

Answer choice (B): Rodriguez will donate her paintings to the museum only if the new wing is named after her. This is a conditional statement. The contrapositve would be if it is not named after her, then she will not donate her paintings, which we would expect to see as the conclusion of the argument. This argument concludes that she will donate her paintings so it does not match the reasoning in the stimulus.

Answer choice (C): The reasoning here does not match the stimulus. The conclusion of the argument is that Reynolds and Khripkova would not make suitable business partners, since they squabble all the time. The argument supports the statement not by conditional reasoning, but by explaining how good business partners solve problems. This does not match the reasoning in the stimulus, so it is incorrect.

Answer choice (D): This is the correct answer choice. If we diagram this argument we have the premise as follows:
  • Sufficient ..... ..... ..... Necessary

    Lopez wins ..... :arrow: ..... Sponsors keep him hydrated
The argument continues and states that since the sponsors do not usually keep their runners hydrated, Lopez will probably not win the marathon. We can diagram this as follows:
  • Sufficient ..... ..... ..... ..... ..... ..... Necessary

    Sponsors keep him hydrated ..... :arrow: ..... Lopez wins marathon
It also contains the two additional aspects of predicting the sponsor’s future actions based on past precedent, and drawing a conclusion of probability, not certainty. This matches the reasoning in the stimulus, so it is correct.

Answer choice (E): This answer choice is phrased in terms of either/or reasoning. There is also conditional reasoning, but it is not necessary to analyze the conditional reasoning because the stimulus did not include either/or. As soon as an answer choice has reasoning that does not match the stimulus, we can eliminate it without further analysis.
 kristenaust
  • Posts: 2
  • Joined: Jun 28, 2019
|
#68044
For answer choice A:

I diagrammed the first sentence as..

if the company downsized ----> NOT more stock issued
if there is more stock issued ----> the company does NOT downsize.

- i thought when there was an "unless" in a premise you flip and negate the 'N'. Am i doing this wrong?
 James Finch
PowerScore Staff
  • PowerScore Staff
  • Posts: 943
  • Joined: Sep 06, 2017
|
#68106
Hi Kristen,

It looks like you have it backwards. The Unless Equation says that we take the condition attached to the "unless" term and make it the necessary, and then negate the other term and make that now-negated term the sufficient condition. In answer choice (A), doing this would make the following diagram:

Downsize :arrow: More Stock Issued

and the contrapositive:

More Stock Issued :arrow: Downsize

So if the company doesn't issue more stock, it must downsize; if it isn't going to downsize, then it must issue more stock. If all we know is that it downsizes or issues more stock, we don't know anything about the other condition.

Hope this clears things up!

Get the most out of your LSAT Prep Plus subscription.

Analyze and track your performance with our Testing and Analytics Package.